Beweis, dass L=S=0L=S=0L=S=0 für gefüllte Elektronenunterschalen?

Es wird allgemein gesagt, dass, wenn Sie eine gefüllte Unterschale haben, wie z P 6 oder D 10 das muss man haben L = S = 0 impliziert J = 0 Und M J = 0 so dass das Atom kugelsymmetrisch ist.

Warum ist das klar L = S = 0 ?

Das ist mir klar M L = M S = 0 . Denn für jedes Elektron mit M l oder M S es gibt ein weiteres Elektron mit M l Und M S So klar M L = ich M l ich = 0 und ebenso für S .

Aber im Allgemeinen, wenn Sie mehrere Drehungen zusammenzählen und finden M L = M S = 0 dies ist KEINE ausreichende Bedingung L = S = 0 .

Dies ist bei dem Zwei-Spin-1/2-Triplettzustand klar, von dem bekannt ist, dass er einen Gesamtspin hat S = 1 :

| ↑↓ + | ↓↑

Ich verlange einen Beweis, dass wir für gefüllte Schalen die obige Eigenschaft erhalten. Ich weiß, dass die Antwort mit der Anforderung der Antisymmetrisierung für die Elektronenwellenfunktionen zusammenhängt. Beispielsweise ist die obige Triplett-Wellenfunktion nicht antisymmetrisch. Wenn ich die antisymmetrische Kombination nehmen würde, hätte ich das Unterhemd, das zufriedenstellend ist S = 0 wie benötigt. Ich möchte jedoch einen Beweis dafür, wie die Anforderung der Antisymmetrisierung dazu führt L = S = 0 für jeden Wert von L Und 2 ( 2 L + 1 ) Elektronen.

Es gibt eine Reihe ähnlicher Fragen auf dieser Seite, die ich bei Bedarf verlinken kann. Keiner von ihnen liefert zufriedenstellende Beweise der Art, wie ich sie mir wünsche. Manchmal weisen sie darauf hin M L = M S = 0 und belasse es dabei. Manchmal weisen sie darauf hin und winken der Antisymmetrisierung zu und nennen es erledigt. Ich hätte gerne etwas Überzeugenderes.

Der Grund, warum mich diese ganze Situation beunruhigt, ist, dass bestimmte Drehimpulszustände für Fermionen und Bosonen auf eine Weise unzugänglich zu sein scheinen, die mir stärker erscheint als das, was durch den Pauli-Ausschluss impliziert wird. Wobei ich vielleicht den Pauli-Ausschluss unterschätze. Ich denke, das sagt aus, wenn ich zum Beispiel 6 habe P Elektronen, dann gibt es nur EINEN Zustand, den sie einnehmen können. Aber die allgemeine Theorie der Drehimpulsaddition besagt, dass diese 6 Spin- 1 / 2 Teilchen mit Spin-1-Bahndrehimpuls sollten etwa 36 Drehimpulszustände haben, die sie einnehmen sollten J = 0 Zu J = 9 .

Offensichtlich denke ich nicht richtig darüber nach. Ich würde es begrüßen, den Beweis zu sehen, nach dem ich suche, und meine Intuition zur Pauli-Antisymmetrisierung klären zu lassen.

Bearbeiten: Die Antwort auf diese Frage Dimension des Hilbert-Raums des Spins 1 / 2 identische Teilchen? spricht meine Intuition zum Pauli-Ausschluss an. Die kurze Antwort lautet: Ja, viele Drehimpulszustände, die für unterscheidbare Teilchen zulässig wären, werden bei der Betrachtung von Fermionen einfach gelöscht. Ich suche noch einen überzeugenden Beweis für Willkür L , das ist nach Anwendung der Antisymmetrisierung der einzige Zustand, der in der verbleibt S = L = J = 0 Zustand.

Wenn M l = M S = 0 , wie nimmt das Atom die z -Achse?
@JEB für allgemeine Drehimpulszustände m = 0 bedeutet nicht, dass der Zustand keine Orientierung hat. Betrachten Sie sphärische Harmonische Y 2 0 ( θ , ϕ ) . Irgendwie macht es die zusätzliche Einschränkung der Antisymmetrisierung so. Das ist die Tatsache, für die ich einen Beweis haben möchte.
M = 0 hat keine Orientierung (eine Vektorrichtung), aber eine Tensorausrichtung in Bezug auf ± z ^ , also wenn M = 0 ...woher sollte das Atom wissen, welche Achse es wählen soll?
@JEB Vielleicht ist es aufgrund einer Wechselwirkung mit einem polarisierten elektrischen Feld in den Zustand geraten, in dem es sich befindet.
Ein Hintergrund E bricht die Symmetrie damit Y l M sind keine Eigenzustände mehr, also gehen Sie nicht dorthin . Der Punkt, den ich gemacht habe, ist, dass wenn M = 0 für eine gefüllte Schale, dann ist das unabhängig davon, wie Sie die Achse wählen, und das gilt nur, wenn L = 0 .
@JEB es hört sich so an, als ob Sie implizieren, dass Atome unter allen Umständen kugelsymmetrisch sein müssen. Das ist falsch. Siehe physical.stackexchange.com/questions/610064/… . Schauen Sie sich einfach ein p-Orbital an (eines davon hat M = 0 ) und Sie werden sehen, dass ein Atom nicht-kugelförmig symmetrisch sein kann, selbst wenn dies der Fall ist M = 0 . Es stimmt, dass es über etwas, das die Symmetrie bricht, dorthin gelangen muss. Aber das symmetriebrechende Element muss kein konstantes Hintergrundfeld sein. es kann ein Lichtimpuls sein, der das verlässt
Atom in einer nichtsphärischen Form. Mein Punkt ist, dass die Form auch dann nicht kugelförmig sein kann M = 0 . Siehe die Y l 0 das ich vorher erwähnt habe. Jedoch aus irgendeinem Grund im Fall der Antisymmetrisierung für Fermionen, mit M S = M L = 0 Zwingt das Atom dazu, sphärisch symmetrisch zu sein, im Gegensatz zum nicht unterscheidbaren Fall. Ich suche den Beweis, warum das so ist, besonders bei Fermionen.

Antworten (3)

Der allgemeine Zustand mit M = 0 Ist:

| L = l , M = 0 = Y l 0 ( θ , ϕ ) = 2 l + 1 4 π P l ( cos θ )

die eine Tensorpolarisation mit einer bevorzugten Ausrichtung entlang der definierenden Achse beschreibt θ . Wenn Sie das auf verschiedene Achsen drehen (gestrichen), dann ist es eine Linearkombination von Kugelflächenfunktionen mit demselben Grad l aber unterschiedliche magnetische Quantenzahlen:

Y l 0 ( θ , ϕ ) = M ' = l l C l M ' Y l M ' ( θ ' , ϕ ' )

das ist kein Zustand mit M ' = 0 , es sei denn l = 0 .

In der Frage geben Sie an, dass Sie wissen, dass ein Atom eine volle Hülle hat M = 0 , sind sich aber unsicher L . Der Punkt ist, dass es keine bevorzugte Achse gibt, also muss eine Achse, die Sie auswählen, vorhanden sein M = 0 , Dann L muss auch Null sein.

Das hat nichts mit dem Pauli-Ausschlussprinzip (PEP) zu tun. Die PEP ergibt sich aus dem Spin-Statistik-Theorem, das besagt, dass die Wellenfunktion identischer Fermionen unter Vertauschung zweier beliebiger Teilchen antisymmetrisch sein muss:

Wenn:

ψ N M ( X 1 , X 2 ) = 1 2 [ ψ N ( X 1 ) ψ M ( X 2 ) ψ M ( X 1 ) ψ N ( X 2 ) ]

Und

ψ N M ( X 2 , X 1 ) = 1 2 [ ψ N ( X 2 ) ψ M ( X 1 ) ψ M ( X 2 ) ψ N ( X 1 ) ] = ψ N M ( X 1 , X 2 )

Wo ( N , M ) Beschriften Sie alle Quantenzahlen, die einen Zustand definieren, dann wenn N = M :

ψ N N ( X 1 , X 2 ) = 0

das ist der PEP. Beachten Sie, dass dies für alle Quantenzahlen gilt, nicht nur L = S = 0 .

Wenn Sie sechs Elektronen in einem einzigen betrachten P -Schale, dann wird die Wellenfunktion durch eine Slater-Determinante beschrieben:

ψ ( 1 , 2 , 3 , 4 , 5 , 6 ) = 1 6 ! ×
| Y 1 1 ( θ 1 , ϕ 1 ) | 1 Y 1 1 ( θ 2 , ϕ 2 ) | 2 Y 1 1 ( θ 3 , ϕ 3 ) | 3 Y 1 1 ( θ 4 , ϕ 4 ) | 4 Y 1 1 ( θ 5 , ϕ 5 ) | 5 Y 1 1 ( θ 6 , ϕ 6 ) | 6 Y 1 0 ( θ 1 , ϕ 1 ) | 1 Y 1 0 ( θ 2 , ϕ 2 ) | 2 Y 1 0 ( θ 3 , ϕ 3 ) | 3 Y 1 0 ( θ 4 , ϕ 4 ) | 4 Y 1 0 ( θ 5 , ϕ 5 ) | 5 Y 1 0 ( θ 6 , ϕ 6 ) | 6 Y 1 1 ( θ 1 , ϕ 1 ) | 1 Y 1 1 ( θ 2 , ϕ 2 ) | 2 Y 1 1 ( θ 3 , ϕ 3 ) | 3 Y 1 1 ( θ 4 , ϕ 4 ) | 4 Y 1 1 ( θ 5 , ϕ 5 ) | 5 Y 1 1 ( θ 6 , ϕ 6 ) | 6 Y 1 1 ( θ 1 , ϕ 1 ) | 1 Y 1 1 ( θ 2 , ϕ 2 ) | 2 Y 1 1 ( θ 3 , ϕ 3 ) | 3 Y 1 1 ( θ 4 , ϕ 4 ) | 4 Y 1 1 ( θ 5 , ϕ 5 ) | 5 Y 1 1 ( θ 6 , ϕ 6 ) | 6 Y 1 0 ( θ 1 , ϕ 1 ) | 1 Y 1 0 ( θ 2 , ϕ 2 ) | 2 Y 1 0 ( θ 3 , ϕ 3 ) | 3 Y 1 0 ( θ 4 , ϕ 4 ) | 4 Y 1 0 ( θ 5 , ϕ 5 ) | 5 Y 1 0 ( θ 6 , ϕ 6 ) | 6 Y 1 1 ( θ 1 , ϕ 1 ) | 1 Y 1 1 ( θ 2 , ϕ 2 ) | 2 Y 1 1 ( θ 3 , ϕ 3 ) | 3 Y 1 1 ( θ 4 , ϕ 4 ) | 4 Y 1 1 ( θ 5 , ϕ 5 ) | 5 Y 1 1 ( θ 6 , ϕ 6 ) | 6 |

Daraus können Sie die Wahrscheinlichkeitsdichte gegen die Winkelkoordinate berechnen, und es sieht so aus:

P ( θ , ϕ ) | Y 1 1 θ , ϕ | 2 + | Y 1 0 θ , ϕ | 2 + | Y 1 1 θ , ϕ | 2
| Sünde θ e + ich ϕ | 2 + | 2 cos θ | 2 + | Sünde θ e ϕ | 2 = 2 ( Sünde 2 θ + cos 2 θ ) = 2

das heißt, es ist kugelsymmetrisch. Sphärische Symmetrie bedeutet L = 0 .

Dies gilt für jede Bestellung:

M = l l | Y l M ( θ , ϕ ) | 2

hängt nicht davon ab θ noch ϕ . Daher sind gefüllte Schalen immer kugelsymmetrisch mit Gesamtdrehimpuls L = 0 .

Dennoch besteht eine tiefe Verbindung zwischen Antisymmetrie und Rotationsinvarianz. Zum Beispiel: das antisymmetrische Levi Civitta-Symbol, ϵ ich J k , ist ein isotroper Tensor. Dies entsteht durch die Schur-Weyl-Dualität, die die rotationsgeschlossenen Unterräume von Tensoren über die Darstellungen der Permutationsgruppe und Young-Diagramme beschreibt. Die Abmessungen der Unterräume lassen sich mit „der bemerkenswerten“ Hakenlängenformel berechnen.

Die antisymmetrische Permutation entspricht einem Unterraum der Dimension 1, der ein Skalar ist (also kugelsymmetrisch). Das einfachste Beispiel ist in Ihrer Frage enthalten, wo Sie zwei 2D-Darstellungen (Spinoren) kombiniert haben und ein symmetrisches Triplett und ein antisymmetrisches Singulett erhalten:

2 2 = 3 S 1 A

Das Triplett transformiert sich wie ein Vektor und das Singulett wie ein Skalar.

Ebenso erhalten Sie, wenn Sie 3 Vektoren (z. B. 3 P-Orbitale) kombinieren:

3 3 3 = 10 S 8 M 8 M 1 A

wo die symmetrisch 10 Ist L = 3 Und L = 1 , die Oktette sind L = 2 Und L = 1 , und das vollständig antisymmetrische Singulett ist L = 0 . Sie können dies überprüfen, indem Sie die Clebsch-Gordon-Koeffizienten mühsam von Hand durcharbeiten.

Ich habe einige Imbissbuden/Fragen. Zunächst zeigen Sie explizit, dass der 6-Elektronen-Zustand die füllt P -Schale ist isotrop. Sie behaupten dann, dass dies für jeden Wert von gilt L . Ich bin gespannt, wie der Beweis dieser letzteren Aussage konkretisiert wird, das ist im Wesentlichen meine ganze Frage. Ihre nächsten Abschnitte über die Beziehung zwischen Antisymmetrie und Rotationsinvarianz scheinen der Schlüssel zu sein. Vor allem, dass die antisymmetrische Permutation Dimension eins ist. Aber offensichtlich sind nicht alle antisymmetrischen Zustände kugelsymmetrisch. Wenn ich nur zwei Elektronen drin habe P -Orbitale...
Sie bilden immer noch einen antisymmetrischen Schiefer-Determinantenzustand, aber er ist nicht rotationssymmetrisch. Es ist also ungefähr so, dass 3 Orbitalzustände und 2 Spinzustände verfügbar sind. Der Einzelteilchen-Hilbert-Raum ist also 6-dimensional. Wenn Sie jetzt 6 Kopien dieses Raums (6 Elektronen) erstellen, sich aber auf antisymmetrische Permutationen beschränken, sagen Sie, dass er eindimensional sein muss. Kann man das leicht beweisen?
Ich denke, der Beweis ist so etwas wie: Da es antisymmetrisch ist, wissen Sie, dass Sie eine Kopie jedes Zustands haben müssen, und es gibt nur eine Möglichkeit, 6 eindeutige Objekte aus einer Gruppe von 6 Objekten auszuwählen.
Wie konnten Sie am Ende der Antwort die Bezeichnungen S, M und A den verschiedenen Unterräumen zuordnen? Ich habe auch nach physical.stackexchange.com/questions/631984/… gefragt , was vielleicht direkter zu dem führt, was ich herauszufinden versuche. Wenn Sie in der Lage sind, könnten Sie vielleicht dort antworten, um zu klären, ob es sinnvoller ist.
Sie erhalten die Symmetrien von etwas namens "The Young Symmetrizer", also haben Sie für jedes Standardtableau ein Rezept, um die rotationsgeschlossenen Permutationen zu erhalten.
Können Sie mehr darüber sagen, wie der Young Symmetrizer verwendet wird, um die Symmetrien zusammengesetzter Drehimpulszustände zu bestimmen? Ich habe versucht, in mich hineinzuschauen, aber es scheint ein wenig verwickelt zu sein. Ich würde mich über eine Antwort hier oder in der anderen Frage freuen, die ich in meinem anderen Kommentar verlinkt habe. Eine gute Referenz wäre auch wünschenswert, wenn dies einfacher zu liefern ist!

Ich möchte eine Antwort teilen, basierend auf dem Beweis (Skizze), den ich in L. Marchildon: Quantum Mechanics gefunden habe. Von den Grundlagen zu Numerischen Methoden und Anwendungen, Kapitel 18 (S. 403f.) .

Da wir uns nur mit gefüllten Unterschalen befassen, definieren wir zunächst den Gesamtspin- und Winkelorbitaloperator für unsere interessierende Unterschale.

L := ich Unterschale L ich , S := ich Unterschale S ich .
Der Index ich läuft über alle Elektronen in dieser Unterschale.

In Bezug auf diese „Gesamt“-Operatoren definieren wir auch Aufwärts- und Abwärtsoperatoren,

L := L X ich L j S := S X ich S j .
Die Erhöhungsoperatoren sind als das hermitesch Konjugierte der Senkungsoperatoren definiert, dh ( L ) = L + = L X + ich L j und ähnlich, ( S ) = S + = S X + ich S j .

Die Wirkung des Absenkoperators L auf einem Quantenzustand Ψ mit Quantenzahlen M L Und L ist zu senken M L um 1, aber behalten L solange fixiert M L > L . Allerdings für minimal M L ( M L = L ) zerstört der Absenkoperator den Zustand Ψ . Analog können wir die Wirkung des Erhöhungsoperators beschreiben L + : Wenn M L < L , es wirkt auf Ψ durch Erhöhung M L um 1 und halten L Fest. Wenn M L ist maximal, dh M L = L , es zerstört den Staat Ψ . (Dasselbe gilt für die Operatoren zur Spinabsenkung und -anhebung bezüglich der Quantenzahlen M S Und S .)
Bisher ist das nichts Neues.


Anmerkung

Lassen Sie mich hier nur anmerken, dass im Allgemeinen Einzelelektronen- Erhöhungs- und -Senkungsoperatoren L ich ± pendeln nicht mit Partikelpermutationen, aber Operatoren zum totalen Anheben und Absenken (z. B. für eine Unterschale) tun es. Das bedeutet, nach dem Handeln mit L ± = ich L ich ± in einem vollständig (anti-)symmetrischen Zustand bleibt es (anti-)symmetrisch. Allerdings ist die Aktion eines einzigen Bedieners L ich ± zerstört (allgemein) jede (Anti-)Symmetrie.

Betrachten Sie zum Beispiel zwei Elektronen (mit den Koordinaten R 1 Und R 2 ) in den Staaten Y l = 1 M = 1 | Und Y l = 1 M = 0 | (Das ist natürlich physikalisch sehr unwahrscheinlich; es dient hier nur zu Demonstrationszwecken.) Ein vollständig antisymmetrisierter Zwei-Elektronen-Zustand ist die Slater-Determinante

Ψ ( R 1 , R 2 ) = Y l = 1 M = 1 ( R 1 ) Y l = 1 M = 0 ( R 2 ) | 1 2 Y l = 1 M = 0 ( R 1 ) Y l = 1 M = 1 ( R 2 ) | 1 2 .
Jetzt handeln mit L An Ψ ( R 1 , R 2 ) wir finden,
L Ψ ( R 1 , R 2 ) = ( L 1 + L 2 ) Ψ ( R 1 , R 2 ) = 2 Y l = 1 M = 0 ( R 1 ) Y l = 1 M = 0 ( R 2 ) | 1 2 2 Y l = 1 M = 1 ( R 1 ) Y l = 1 M = 1 ( R 2 ) | 1 2 + 2 Y l = 1 M = 1 ( R 1 ) Y l = 1 M = 1 ( R 2 ) | 1 2 2 Y l = 1 M = 0 ( R 1 ) Y l = 1 M = 0 ( R 2 ) | 1 2 .
(Das Aussehen des Faktors 2 ist aus der Identität ersichtlich L L + = L 2 L z 2 L z .) Dieser Zustand ist eindeutig wieder antisymmetrisch, wenn auch keine Slater-Determinante.

Man kann verifizieren, dass ein allgemeiner antisymmetrischer Zustand – der als Summe von Slater-Determinanten geschrieben werden kann – nach der Wirkung von antisymmetrisch bleibt L ± . (Wenn Sie diese Aussage für eine Slater-Determinante glauben, dann verwenden Sie einfach, dass die Partikelpermutation eine lineare Operation ist.)


Betrachten Sie nun einen Zustand Ψ mit Eigenwerten L , M L , S , M S in Bezug auf die gesamten Subshell-Operatoren, die wir oben definiert haben. Außerdem, Ψ sollte eine gefüllte Unterschale beschreiben. Im Folgenden werden wir das beweisen M L = L = 0 Und M S = S = 0 wie.

Vereinfachen wir die Situation etwas und betrachten zunächst den Drehimpuls L Und M L und bedenke, was passiert, wenn wir mit handeln L An Ψ . Durch def. von L , ist die Aktion des Subshell-Absenkungsoperators nur die Summe der Absenkungsoperatoren L ich für jedes Elektron ich ,

L = ich Unterschale L ich .
(Wie wir gesehen haben, ist das Bilden der Summe wichtig, um das Ergebnis antisymmetrisiert zu halten).

Seit Ψ ein vollständig antisymmetrisierter Zustand ist, kann er mehrere Terme/Summanden enthalten. Wir verwenden eine Eigenbasis der L ich (oder eher L ich 2 Und L ich z ) als Ein-Teilchen-Basis, in Bezug auf die wir antisymmetrisieren – siehe Beispiel in der obigen Bemerkung. Das heißt, auf jedem Summanden der Operator L ich wirkt auf den Zustand des Elektrons ich mit einer wohldefinierten Quantenzahl M l ich .

  • Wenn M l ich > l ich , Dann L ich senkt sich einfach M l ich M l ich 1 . Da die Unterschale aber voll besetzt ist, nimmt bereits ein anderes Elektron diesen Zustand ein. Wir haben also zwei Elektronen im gleichen Zustand. Agieren mit dem Total Subshell Senkoperator L An Ψ sorgt für Antisymmetrie des gesamten Endzustandes L Ψ , wie wir in der obigen Bemerkung gesehen haben. Die Antisymmetrierung eines Zustands mit zwei Elektronen im selben Zustand ergibt Null (Pauli-Ausschlussprinzip).
  • Wenn M l ich = l ich , dh M l ich minimal ist, können wir die Quantenzahl nicht verringern M l ich alle weiteren und diesen Begriff / Summanden von Ψ ist zerstört.

Insgesamt finden wir L Ψ = 0 .

Durch eine analoge Argumentation - unter Verwendung der Eigenschaften des vollständigen Subshell-Zustands Ψ beim Handeln mit L + – können wir ableiten L + Ψ = 0 .

Jetzt kommt der Trick (man ahnt vielleicht schon, was als nächstes kommt): Denn sowohl der Aufwärts- als auch der Abwärtsoperator zerstören Ψ , seine Quantenzahl M L muss gleichzeitig minimal und maximal sein . Das funktioniert nur, wenn L = 0 was einschränkt M L auch null sein.

Am Anfang habe ich auch einen Hebe- und Senkoperator definiert S ± für den Spin. Obwohl ich die Details hier nicht gebe, ist der Beweis für S = M S = 0 funktioniert genauso wie für den Drehimpuls, den wir hier gemacht haben.

Ein anderer Ansatz

(Verwandt mit der Antwort von JEB)

Wenn Sie glauben, dass die leere Hüllen haben L = S = 0 , dann können Sie sich durch eine Partikel-Loch-Transformation davon überzeugen, dass gefüllte Schalen genau die gleiche Eigenschaft haben. Grundsätzlich läuft dies darauf hinaus, die Elektronenerzeugung auf Lochvernichtungsoperatoren (und die Elektronenvernichtung auf Locherzeugungsoperatoren) abzubilden, während dem entsprechenden Loch auch der umgekehrte magnetische Drehimpuls und die Spinquantenzahl als dem Elektron zugewiesen werden, dh l l , M l M l , S S , M S M S . Diese Zuordnung gewährleistet die (Matrix-)Darstellung der Operatoren L Und S werden durch die Transformation nicht verändert, sodass alle Eigenschaften bzgl L Und S die Anwendung auf elektronische Leerhülsen gilt auch für gefüllte Lochhülsen. (Die Details dieser Transformation sind auch in Lehrbüchern der Atomphysik und Online-Vorlesungsmitschriften zu finden.)

Dies hängt natürlich mit der Tatsache zusammen, die von JEB näher ausgeführt wurde: Jedes Objekt, das invariant ist unter Rotationen, dh Aktionen unter einer Gruppe, die von generiert wird L , transformiert sich als Tensor vom Rang Null, also als skalare Größe in unserem Beispiel von Drehimpuls und Raumrotationen. Dies ist in der Tat unabhängig vom Pauli-Ausschlussprinzip und dem Raum, in dem die Gruppenrepräsentation agiert.

KONTEXT

Die Frage hier verlangt nach einem Beweis , dass für eine gefüllte Unterschale der Gesamtdrehimpuls L , und Gesamtspin-Drehimpuls, S , sind solche

L = S = 0 .

Oben gibt @JEB eine Antwort auf die Frage. Ich habe keine Probleme mit dieser Antwort, außer dass (soweit ich das beurteilen kann) nichts erwähnt wird S . Daher bleibt, soweit ich das beurteilen kann, die ursprüngliche Frage unbeantwortet. in meiner Antwort liefere ich keinen Beweis , der verlangt wird; vielmehr liefere ich eine Demonstration, die meine Vernunft anzeigt. Ich stelle mir vor, dass eine solche Begründung zu einem Beweis erweitert werden könnte.

DEMONSTRATION

Magnesium ( Z = 12 ) ist ein zweiwertiges Metall mit der elektronischen Grundzustandskonfiguration [Ne]3s 2 . Wie groß ist der Drehimpuls des Grundzustands?

Im LS-Schema ( https://en.wikipedia.org/wiki/Angular_momentum_coupling ) lauten die Regeln

L [ N e ] 3 S 2 = 1 + 2   Und S [ N e ] 3 S 2 = S 1 + S 2 .

Für die 1s 2 ,2 Sek 2 , 2p 6 ,3s 2 Grundzustand haben wir zwei Valenzelektronen im S Hülse. Jedes einzelne Elektron in der S Shell hat die Quantenzahl = 0 . Deshalb 1 = 2 = 0 . Damit sind die möglichen Werte der Quantenzahlen für

L [ N e ] 3 S 2 = 1 + 2   mit   1 = 0 Und 2 = 0 ,
Sind
L [ N e ] 3 S 2 = { ( 0 + 0 ) , , ( | 0 0 | ) } = { 0 } .
Somit,
| L [ N e ] 3 S 2 | = 0 ( 1 + 0 ) = 0 ,
Und
L [ N e ] 3 S 2 = 0 .

Für die 1s 2 ,2 Sek 2 , 2p 6 ,3s 2 Grundzustand haben wir zwei Valenzelektronen im S Hülse. Jedes Elektron hat die Quantenzahl S = 1 / 2 . Deshalb S 1 = S 2 = 1 / 2 . Damit sind die möglichen Werte der Quantenzahlen für

S [ N e ] 3 S 2 = S 1 + S 2   mit   S 1 = 1 / 2 Und S 2 = 1 / 2 ,
zu sein scheinen
S [ N e ] 3 S 2 = { ( 1 / 2 + 1 / 2 ) , , ( | 1 / 2 1 / 2 | ) } = { 1 , 0 } .
Es scheint weiter so
| S [ N e ] 3 S 2 | = { 0 ( 1 + 0 ) , 1 ( 1 + 1 ) } .
Dabei müssen wir folgendes berücksichtigen.

„Im Spezialfall von Grundkonfigurationen äquivalenter Elektronen folgen der Spin und der Bahndrehimpuls des Terms mit der niedrigsten Energie einigen halbkreisförmigen Regeln, die als Hundsche Regeln bezeichnet werden: Der Term mit der niedrigsten Energie hat den größten Wert von S im Einklang mit dem Pauli-Ausschlussprinzip. (Atomic Physics, Foot, 2005, S.81)''

In diesem Beispiel haben wir es eindeutig mit der Grundkonfiguration und dem Term mit der niedrigsten Energie zu tun. Nach dem Pauli-Ausschlussprinzip können wir im Quantenzustand keine zwei Elektronen haben

| N , , M , S , M S >= | 3 , 0 , 0 , 1 / 2 , + 1 / 2 > ;
ebenso können wir im Quantenzustand keine zwei Elektronen haben
| N , , M , S , M S >= | 3 , 0 , 0 , 1 / 2 , 1 / 2 > .
Somit muss für die Grundkonfiguration jeweils eines (aber nicht beide) der Elektronen wie folgt quantisiert werden:
| ψ 1 >= | N , , M , S , M S >= | 3 , 0 , 0 , 1 / 2 , + 1 / 2 > | ψ 2 >= | N , , M , S , M S >= | 3 , 0 , 0 , 1 / 2 , 1 / 2 >
Der Wert von S = 1 , was zu einem Wert ungleich Null führen würde | S [ N e ] 3 S 2 | , ist physikalisch nicht realisierbar, da wir für den Spin zwei Elektronen benötigen würden, deren einzelne Spindrehimpulse ungleich und entgegengesetzt sind. Dies wäre mit dem Pauli-Ausschlussprinzip (dh die beiden Spindrehimpulse müssen gleich und entgegengesetzt sein und ergeben somit keinen Netto-Spindrehimpuls) vereinbar. Gemäß der Hundschen Regel finden wir das
S [ N e ] 3 S 2 = { 0 } .
Somit,
| S [ N e ] 3 S 2 | = 0 ( 1 + 0 ) = 0 ,
Und
S [ N e ] 3 S 2 = 0 .

Zusammenfassend habe ich einen bestimmten Fall demonstriert, bei dem für eine gefüllte Unterschale der Gesamtdrehimpuls L , und Gesamtspin-Drehimpuls, S , sind identisch Null. Ich finde nämlich

L [ N e ] 3 S 2 = S [ N e ] 3 S 2 = 0 .